LSAT and Law School Admissions Forum

Get expert LSAT preparation and law school admissions advice from PowerScore Test Preparation.

 Administrator
PowerScore Staff
  • PowerScore Staff
  • Posts: 8917
  • Joined: Feb 02, 2011
|
#41630
This question asks you to determine what must be true if N is presented sixth. Since N is not given by K, we need to ensure that K gives the first, third, and fifth presentations in compliance with the first rule. See Template 1.
PT65_D11 LG Explanations_game_#2_#10_diagram 1.png
Next, recall that two of K’s presentations—S and T—are each subject to a sequencing rule involving O and W, respectively. So, if N is delivered sixth, then neither S nor T can be delivered fifth. Therefore, the subject of the fifth presentation must be P:
PT65_D11 LG Explanations_game_#2_#10_diagram 2.png
Considering this is a Must Be True question, you should immediately check if your inference about P is sufficient to validate any of the answer choices. And indeed, that inference immediately proves answer choice (B) to be correct. Note that the dual options in our local setup (above) are given for informational purposes only, and need not be part of your diagram.

Answer choice (A) is incorrect, because O could also be presented second.

Answer choice (B) is the correct answer choice, as explained above.

Answer choice (C) is incorrect, because S could also be presented first.

Answer choice (D) is incorrect, because T could also be presented third.

Answer choice (E) is incorrect, because W could also be presented second.
You do not have the required permissions to view the files attached to this post.

Get the most out of your LSAT Prep Plus subscription.

Analyze and track your performance with our Testing and Analytics Package.